Está en la página 1de 6

UPSC Civil Services Main 2000 - Mathematics

Calculus
Sunder Lal
Retired Professor of Mathematics
Panjab University
Chandigarh

January 2, 2010

Question 1(a) Use the mean value theorem to prove that


2 2
log 1.4
7 5
Solution. Let f (x) = log(1 + x). Consider the interval [0, x]. Clearly the mean value
1
theorem applies: log(1 + x) log 1 = 1+ x for some [0, x]. Take x = 0.4.

2
0 0.4 =
5
7
1 1+
5
5 1
x xx
7 1+
2 2
log 1.4
7 5

Question 1(b) Show that


ZZ
1 (l)(m)
x2l1 y 2m1 dx dy = r2(l+m)
A 4 (l + m + 1)

where A consists of all positive values of x, y lying insode the circle x2 + y 2 = r2 .

1
Solution. Switching to polar coordinates, x = R cos , y = R sin and
Z rZ
2
I = R2l1 cos2l1 R2m1 sin2m1 R d dR
0 0
Z r Z
2
2l+2m1
= R dR cos2l1 sin2m1 d
0 0
r 2l+2m ( 2 )( 2m
2l
2
)
= 2l1 2m1
2l + 2m 2( 2 + 2 + 1)
r2(l+m) (l)(m) 1 (l)(m)
= = r2(l+m)
4(l + m) (l + m) 4 (l + m + 1)

x2 y 2 z 2
Question 2(a) Find the center of gravity of the positive octant of the ellipsoid 2 + 2 + 2 =
a b c
1, if the density varies as xyz.

Solution. ZZZ
M ass = x2 2 2 xyz dx dy dz
+ yb2 + zc2 1
a2
x 0, y 0, z 0
Let x = aX, y = bY, z = cZ. We get:
ZZZ
2 2 2
M ass = a b c XY Z dX dY dZ
X2 + Y 2 + Z2 1
X 0, Y 0, Z 0
Z 1 Z 1x2 Z 1x2 y2
= a2 b 2 c 2 x y z dz dy dx
0 0 0

2 2 2 Z 1 Z 1x2
abc
= x y(1 x2 y 2 ) dy dx
2 0 0
1 1x2
a2 b 2 c 2 y 2 x2 y 2 y 4
Z 
= x dx
2 0 2 2 4 0
a2 b 2 c 2 1
Z
x 2(1 x2 ) 2x2 (1 x2 ) (1 x2 )2 dx

=
8 0
2 2 2 Z 1
abc
= (x 2x3 + x5 ) dx
8 0
2 2 2
 2 1
abc x x4 x6
= 2 +
8 2 4 6 0
2 2 2
a2 b 2 c 2
 
abc 1 1 1
= + =
8 2 2 6 48

2
If (x, y, z) are the coordinates of the center of gravity, then
ZZZ
M ass x = x2 y2 z2 x2 yz dx dy dz
a2 + b2 + c 2 1
x 0, y 0, z 0
ZZZ
3 2 2
= abc X 2 Y Z dX dY dZ
X2 + Y 2 + Z2 1
X 0, Y 0, Z 0
3 2 2 Z 1
abc
= (x2 2x4 + x6 ) dx
8
0
a3 b 2 c 2 1 2 1 a3 b 2 c 2

= + =
8 3 5 7 105

a3 b 2 c 2
 
48 16 16a 16b 16c
Thus x = 2 2 2 = a. By symmetry, the center of gravity is , , .
105 abc 35 35 35 35

(
0, x irrational
Question 2(b) Let f (x) =
1, x rational
Show that f is not integrable on [a, b].

Solution. Let P = {a = x0 < x1 < x2 < . . . xn = b} be any partition of [a, b]. In any
interval [xi1 , xi ], 1 i n, there exist rationals as well as irrationals.
Thus mi = minxi1 xxi f (x) = 0, Mi = max Pn xi1 xxi f (x) = 1.
The Lower Riemann Pn Sum = L(P, f ) = i=1 mi (xi xi1 ) = 0. The Upper Riemann
Sum = U (P, f ) = i=1 Mi (xi xi1 ) = b a.
Z b Z b
Thus f (x) dx = 0 and f (x) dx = b a, showing that f is not Riemann integrable
a a
on [a, b].

Question 2(c) Show that

dn log x
   
n n! 1 1 1
= (1) n+1 log x 1 . . .
dxn x x 2 3 n

Solution. We can prove this by induction. It is obviously true for n = 0. Suppose it is


true for n :
dn log x
   
n n! 1 1 1
= (1) n+1 log x 1 . . .
dxn x x 2 3 n

3
Differentiating both sides, we get

dn+1 log x
    
n d 1 1 1 1
= (1) n! log x 1 . . .
dxn+1 x dx xn+1 2 3 n
    
n (n + 1) 1 1 1 1 1
= (1) n! n+2
log x 1 . . . + n+1
x 2 3 n x x
   
n+1 1 1 1 1 1
= (1) (n + 1)! n+2 log x 1 . . . n+2
x 2 3 n x (n + 1)
 
(n + 1)! 1 1 1 1
= (1)n+1 n+2 log x 1 . . .
x 2 3 n n+1

which proves the result for n + 1, and thus by induction for all n.
An alternate method is to use the Leibniz rule:
n   nr
dn X n d u dr v
(uv) =
dxn r=0
r dxnr dxr

and letting u = x1 , v = log x.

Question 2(d) Find constants a, b which minimize


Z
2
F (a, b) = sin x (ax2 + bx) dx
0

Solution.
Z
sin2 x 2(ax2 + bx) sin x + (ax2 + bx)2 dx

F (a, b) =
Z0  
1 cos 2x
Z
x sin 2x
sin2 x dx = dx = =
0 0 2 2 4 0 2
Z  Z
(ax2 + bx) sin x dx = (ax2 + bx) cos x + (2ax + b) cos x dx
0 0 0
  Z
2
= (a + b) + (2ax + b) sin x 2a sin x dx
0 0

= (a + b) + 2a cos x = a 2 + b 4a
2

0

a2 5 ab 4 b2 3
Z
(a2 x4 + 2abx3 + b2 x2 ) dx = + +
0 5 2 3
a2 5 ab 4 b2 3
F (a, b) = 2(a 2 + b 4a) + + +
2 5 2 3
1h 5 2 i
= 6 a + 15 4 ab + 10 3 b2 (60 2 240)a 60b + 15
30

4
Let G(a, b) = 30F (a, b). Then at the maximum:
G
= 12 5 + 15 4 b (60 2 240) = 0
x
G
= 15 4 a 20 3 b 60 = 0
y
240 12 2 20 2 320
Solving these for a, b we get b = , a = .
4 5  2 2
2G 5 2G 3 2G 4 2G 2G G
Now = 12 > 0, = 20 > 0, = 15 =
a2 b2 a b a2 b2 a b
12 5 20 3 (15 4 )2 = 15 8 > 0. Thus G(a, b) and hence F (a, b) is minimum when
240 12 2 20 2 320
b= , a = .
4 5
Paper II

Question 3(a) 1. Suppose f is real-valued twice differentiable on (0, ) and M0 , M1 , M2


the least upper bounds of |f (x)|, |f 0 (x)|, |f 00 (x)| respectively in (0, ). Prove that for
each x > 0, h > 0
f (x + 2h) f (x)
f 0 (x) = hf 00 (u)
2h
for some u (x, x + 2h). Hence show that M12 4M0 M2 .
ZZ
2. Evaluate (x3 dy dz + x2 y dz dx + x2 z dx dy) by transforming it into a triple integral
S
where S is the closed surface formed by the cylinder x2 + y 2 = a2 , 0 z b and the
circular discs x2 + y 2 a2 , z = 0 and x2 + y 2 a2 , z = b.

Solution.
1. Clearly f (x) satisfies the requirements of Taylors theorem in [x, x + 2h]. Therefore
2
f (x + 2h) = f (x) + 2hf 0 (x) + (2h)
2!
f 00 (u) for some u (x, x + 2h). Thus

f (x + 2h) f (x)
f 0 (x) = hf 00 (u)
2h
as required.
Now
|f (x + 2h)| + |f (x)| M0 + M0 M0
|f 0 (x)| + h|f 00 (u)| + hM2 = + hM2
2h 2h h
for all x and h > 0. Therefore
 
0 M0
sup |f (x)| inf + hM2
x>0 h>0 h

5
2

q q
M0 M0
But h
+ hM2 = h
M2 h + 2 M0 M2 , so for h = M 0
M2
, we have M0
h
+ hM2 =

 
M0
2 M0 M2 inf h>0 h
+ hM2 2 M0 M2 . Thus

 
0 M0 p
M1 = sup |f (x)| inf + hM2 2 M0 M2
x>0 h>0 h

so M12 4M0 M2 .

2. By Greens theorem
ZZ
(x3 dy dz + x2 y dz dx + x2 z dx dy)
Z ZSZ  
3 2 2
= x + x y + x z dx dy dz
x2 +y 2 a2 x y z
0zb
ZZ Z b  ZZ
2
= 5x dz dy dx = 5b x2 dy dx
x2 +y 2 a2 0 x2 +y 2 a2
Z a Z 2 4
a 5
= 5b r2 cos2 r d dr = 5b = a4 b
0 0 4 4

También podría gustarte